Những câu hỏi liên quan
Traan Dungx
Xem chi tiết
Cam Ngoc Tu Minh
12 tháng 8 2023 lúc 11:08

 

Ta có:

P = a + b + c a + b + a + b = 2(a + b) 2(-1) = -2

Ta cũng có:

P = a + b + c a + b + c - 2abc a + b + c - 2(-1)(-1)(-1) = -3

Vậy GTNN của P = -3 và GTLN của P = -2.

Bình luận (0)
minhduc
Xem chi tiết
Phí Thúy Nga
29 tháng 6 2021 lúc 10:28

12632t54s jsd

Bình luận (0)
 Khách vãng lai đã xóa
Tạ Uyên
Xem chi tiết
Nguyễn Việt Lâm
8 tháng 12 2021 lúc 19:49

Do \(a^2+b^2+c^2=1\Rightarrow0\le a;b;c\le1\)

\(\Rightarrow\left\{{}\begin{matrix}\left(a-1\right)\left(b-1\right)\left(c-1\right)\le0\\b^{2011}\le b\\c^{2011}\le c\end{matrix}\right.\)

\(\Rightarrow T\le a+b+c-ab-bc-ca=\left(a-1\right)\left(b-1\right)\left(c-1\right)+1-abc\le1-abc\le1\)

\(T_{max}=1\) khi \(\left(a;b;c\right)=\left(0;0;1\right)\) và các hoán vị

Bình luận (0)
minhduc
Xem chi tiết
minhduc
26 tháng 10 2017 lúc 18:24

Bài 8:

Cho các số thực a,b,c,x,y thỏa mãn ax−by=√3ax−by=3.

Tìm GTNN của F=a2+b2+x2+y2+bx+ayF=a2+b2+x2+y2+bx+ay

Lời giải:

Sử dụng giả thiết ax−by=√3ax−by=3 ta có:

(a2+b2)(x2+y2)=(ax+by)2+(ax−by)2=(ax+by)2+3(a2+b2)(x2+y2)=(ax+by)2+(ax−by)2=(ax+by)2+3

Áp dụng bất đẳng thức CauchyCauchy , suy ra:

a2+b2=x2+y2=(a2+b2)+(x2+y2)≥2√(a2+b2)(x2+y2)=2√(ax+by)2+3a2+b2=x2+y2=(a2+b2)+(x2+y2)≥2(a2+b2)(x2+y2)=2(ax+by)2+3

Do đó, ta đưa về bài toán tìm GTNN của: 2√x2+3+x2x2+3+x trong đó x=ax+byx=ax+by

Ta có:

(2√x2+3+x)2=4(x2+3)+4x√x2+3+x2=(x2+3)+4x√x2+3+4x2+9=(√x2+3+2x)2+9≥9(2x2+3+x)2=4(x2+3)+4xx2+3+x2=(x2+3)+4xx2+3+4x2+9=(x2+3+2x)2+9≥9

⇒2√x2+3+x≥3⇒2x2+3+x≥3

Vậy MinT=3MinT=3

Bài 11:Cho các số a,b,c không âm không đồng thời bằng không. Chứng minh rằng;

∑2a2−bcb2−bc+c2≥3∑2a2−bcb2−bc+c2≥3

Không mất tính tổng quát, ta có thể giả sử bb là số nằm giữa aa và cc

BĐT đã cho tương đương với

∑2a2+(b−c)2b2−bc+c2≥6∑2a2+(b−c)2b2−bc+c2≥6

Áp dụng BĐT Cauchy-Schwarz, ta có

∑2a2b2−bc+c2≥2(a2+b2+c2)2∑a2(b2−bc+c2)=2(a2+b2+c2)22∑a2b2−abc∑a∑2a2b2−bc+c2≥2(a2+b2+c2)2∑a2(b2−bc+c2)=2(a2+b2+c2)22∑a2b2−abc∑a

∑(b−c)2b2−bc+c2≥[a(b−c)+b(a−c)+c(a−b)]22∑a2b2−abc∑a=4b2(a−c)22∑a2b2−abc∑a∑(b−c)2b2−bc+c2≥[a(b−c)+b(a−c)+c(a−b)]22∑a2b2−abc∑a=4b2(a−c)22∑a2b2−abc∑a

Do đó ta chỉ cần chứng minh

(a2+b2+c2)2+2b2(a−c)2≥6∑a2b2−3abc∑a(1)(a2+b2+c2)2+2b2(a−c)2≥6∑a2b2−3abc∑a(1)

Ta có 

b2(a−c)2=[a(b−c)+c(a−b)]2=a2(b−c)2+c2(a−b)2+2ac(a−b)(b−c)b2(a−c)2=[a(b−c)+c(a−b)]2=a2(b−c)2+c2(a−b)2+2ac(a−b)(b−c)

≥a2(b−c)2+c2(a−b)2≥a2(b−c)2+c2(a−b)2

Suy ra 

2b2(a−c)2≥a2(b−c)2+b2(c−a)2+c2(a−b)22b2(a−c)2≥a2(b−c)2+b2(c−a)2+c2(a−b)2

⇒VT(1)≥(∑a2)2+2∑a2b2−2abc∑a⇒VT(1)≥(∑a2)2+2∑a2b2−2abc∑a

Do đó ta chỉ còn phải chứng minh 

(∑a2)2+2∑a2b2−2abc∑a≥6∑a2b2−3abc∑a(∑a2)2+2∑a2b2−2abc∑a≥6∑a2b2−3abc∑a

⇔∑a4+abc∑a≥2∑a2b2⇔∑a4+abc∑a≥2∑a2b2

BĐT này hiển nhiên đúng theo BĐT Schur

∑a4+abc∑a≥∑ab(a2+b2)∑a4+abc∑a≥∑ab(a2+b2)

Và BĐT AM-GM

∑ab(a2+b2)≥2∑a2b2∑ab(a2+b2)≥2∑a2b2

Kết thúc chứng minh 

Đẳng thức xảy ra khi a=b=ca=b=c hoặc a=ba=b, c=0c=0 và các hoán vị.

Bình luận (0)
Sakuraba Laura
26 tháng 10 2017 lúc 18:27

Bạn leminhduc tự hỏi tự trả lời à

Bình luận (0)
Võ Quang Nhân
Xem chi tiết
Nguyễn Thị Minh Nhã
22 tháng 5 2022 lúc 19:42

P≤a2+2aab+2b2+b2+22bc+2c2+c2+22ca+2a2

P≤(a+2b)2+(b+2c)2+(c+2a)2

Bình luận (0)
hoàng minh chính
Xem chi tiết
Đặng Gia Ân
Xem chi tiết
Phạm Ngọc Bích
17 tháng 1 2022 lúc 16:23
Ngu kkkkkkkkkkkkkkkkkkkkkkkkkkkkkk
Bình luận (0)
 Khách vãng lai đã xóa
Huỳnh Ngọc Lam
Xem chi tiết
Nguyễn Việt Lâm
26 tháng 1 2022 lúc 8:01

\(P=\dfrac{a^2+b^2+c^2}{ab+bc+ca}\ge\dfrac{ab+bc+ca}{ab+bc+ca}=1\)

\(P_{min}=1\) khi \(a=b=c=1\)

\(P=\dfrac{\left(a+b+c\right)^2-2\left(ab+bc+ca\right)}{ab+bc+ca}=\dfrac{9}{ab+bc+ca}-2\)

Do \(a;b\ge1\Rightarrow\left(a-1\right)\left(b-1\right)\ge0\Rightarrow ab\ge a+b-1=2-c\)

\(\Rightarrow ab+c\left(a+b\right)\ge2-c+c\left(3-c\right)=-c^2+2c+2=c\left(2-c\right)+2\ge2\)

\(\Rightarrow P\le\dfrac{9}{2}-2=\dfrac{5}{2}\)

\(P_{max}=\dfrac{5}{2}\) khi \(\left(a;b;c\right)=\left(1;2;0\right);\left(2;1;0\right)\)

Bình luận (0)
Vũ Thu Hiền
Xem chi tiết
Akai Haruma
30 tháng 1 2021 lúc 22:04

Lời giải:

Do $b\leq c; a^2\geq 0$ nên $a^2(b-c)\leq 0$

$\Rightarrow Q\leq b^2(c-b)+c^2(1-c)$

Áp dụng BĐT AM-GM:

\(b^2(c-b)=4.\frac{b}{2}.\frac{b}{2}(c-b)\leq 4\left(\frac{\frac{b}{2}+\frac{b}{2}+c-b}{3}\right)^3=\frac{4}{27}c^3\)

\(\Rightarrow Q\leq c^2-\frac{23}{27}c^3=c^2(1-\frac{23}{27}c)=(\frac{54}{23})^2.\frac{23}{54}c.\frac{23}{54}c(1-\frac{23}{27}c)\leq (\frac{54}{23})^2\left(\frac{\frac{23}{54}c+\frac{23}{54}c+1-\frac{23}{27}c}{3}\right)^3=\frac{108}{529}\)

Vậy $Q_{max}=\frac{108}{529}$

Giá trị này đạt tại $(a,b,c)=(0,\frac{12}{23}, \frac{18}{23})$

Bình luận (0)
Akai Haruma
30 tháng 1 2021 lúc 22:04

Lời giải:

Do $b\leq c; a^2\geq 0$ nên $a^2(b-c)\leq 0$

$\Rightarrow Q\leq b^2(c-b)+c^2(1-c)$

Áp dụng BĐT AM-GM:

\(b^2(c-b)=4.\frac{b}{2}.\frac{b}{2}(c-b)\leq 4\left(\frac{\frac{b}{2}+\frac{b}{2}+c-b}{3}\right)^3=\frac{4}{27}c^3\)

\(\Rightarrow Q\leq c^2-\frac{23}{27}c^3=c^2(1-\frac{23}{27}c)=(\frac{54}{23})^2.\frac{23}{54}c.\frac{23}{54}c(1-\frac{23}{27}c)\leq (\frac{54}{23})^2\left(\frac{\frac{23}{54}c+\frac{23}{54}c+1-\frac{23}{27}c}{3}\right)^3=\frac{108}{529}\)

Vậy $Q_{max}=\frac{108}{529}$

Giá trị này đạt tại $(a,b,c)=(0,\frac{12}{23}, \frac{18}{23})$

Bình luận (0)